LSAT and Law School Admissions Forum

Get expert LSAT preparation and law school admissions advice from PowerScore Test Preparation.

User avatar
 Dave Killoran
PowerScore Staff
  • PowerScore Staff
  • Posts: 5852
  • Joined: Mar 25, 2011
|
#26537
Complete Question Explanation

The correct answer choice is (B)

The best strategy on List questions is to use the individual rules of the game to eliminate incorrect answer choices.

Answer choice (A) can be eliminated because F must be second (and cannot be first).

Answer choice (C) can be eliminated because J cannot be seventh.

Answer choice (D) can be eliminated because G and H cannot be consecutive.

Answer choice (E) can be eliminated because H must be in some track before L.

Answer choice (B) is thus proven correct by process of elimination.

Get the most out of your LSAT Prep Plus subscription.

Analyze and track your performance with our Testing and Analytics Package.